Practice Quantitative Aptitude Questions For IBPS 2017 Exams (Simplification and Data Sufficiency)

Practice Quantitative Aptitude Questions For IBPS 2017 Exams (Simplification and Data Sufficiency):

Dear Readers, Important Practice Aptitude Questions for IBPS Exams 2017 was given here with Solutions. Aspirants those who are preparing for the Bank Examination and other Competitive Examination can use this material.

1). (468 ÷ 78 × 26)/16 = ?2 + (35 ÷ 500×2)
a)  9.61
b)  10
c)  3.1
d)  0.31
e)  None of these
2). 

 

a)  272
b)  13.5
c)  142
d)  182.25
e)  (19.6)2
3). 32 % of 600 × 6 1/3 ÷ 4 2/9 = ?
a)  264
b)  288
c)  304
d)  328
e)  None of these
4). (54 ÷ 0.6 ÷ 0.12) ÷ (30% of ?) = 9 1/3 ÷ 11 1/5
a)  3000
b)  650
c)  4500
d)  2300
e)  None of these
5). 

 

a)  0.3
b)  2.3
c)  140
d)  140.3
e)  None of these
Direction (Q.6-10):  Each of the questions below consists of a question and two statements numbered I and II given below it. You have to decide whether the data given in the statements are sufficient to answer the question. Read both statement and choose the most appropriate option.
a)    The data even in both statements I and II together are not sufficient to answer the question.
b)    The data in both statements I and II together are necessary to answer the question. 
c)   The data in statement II alone are sufficient to answer the question, while the data in statement I alone are not sufficient to answer the question.
d)    The data either in statement I alone or in statement II alone are sufficient to the answer the question.
e)   The data in statement I alone are sufficient to answer the question while the data in statement II alone are not sufficient to answer the question. 
6). What is the present age of C?
I. At present A is 8 years elder to B. C is 2 years younger than A.
II. The ratio between present age of B and that of C is 3:4.
7). What was the annual salary of the Mrs.Santhi in 2016?
I. Out of the annual salary in 2015, Mrs.Santhi invested 19% in stocks and 21% in mutual funds and 45% of her annual salary on household expenses and save an amount of Rs. 3 lakh at end of the year.
II. Annual salary of Mrs.Santhi increased by 8% from 2015 to 2016 and this got a raise of Rs.16,000 in her annual salary.
8). What is the speed of the train?
I. The train crosses a vertical pole in 6 seconds.
II. The train crosses a 120m long platform in 10 seconds while travelling at the same speed.
9). What is the present age of Anand’s mother?
I. The respective ratio between the present ages of Anand and his mother is 6:11.
II. Anand’s mother is 5 years younger than his father.
10). What is the number?
I. 25% of that number is 40% of 280

 

II. Three- fourths of that number is less than that number by 112. 

Explanation:
1). Answer: c)
2). Answer: d)
3). Answer: b)
32 % of 600 × 6 1/3 ÷ 4 2/9 = ?
32/100 × 600 × 19/3 × 9/38 = ?
∴ ? = 288
 
4). Answer: a)
(54 ÷ 0.6 ÷ 0.12)÷ (30% of ?) = 9 1/3 ÷ 11 1/5
54/(0.6 ×0.12) ÷ (30/100 × ?) = 28/3 × 5/56
750 × 10/3 × 1/? = 28/3 × 5/56           
∴ ? = (750×10×3×56)/(3×28×5)
   Hence, ? = 3000
 
5). Answer: c)
6). Answer: b)
From statement I& II,
Let present age of ‘A’ be x
∴ B’s present age is (x-8)
  C’s present age is (x-2)
  (x-8)/(x-2) = 3/4
  4x-32 = 3x-6
  x =26
Hence, the present age of C is 26-2=24 years
Hence, both statements I& II are sufficient to answer.
 
7). Answer: c)
From statement-I,
Santhi’s annual salary be ‘x’
Total expenditure= (19+21+45)= 85
∴ 100 – 85 = 15% of salary = 3 lakh
∴ x = (300000 × 100)/15 = 20 lakh
From statement-II,
8% of salary is16,000
∴ x = 16,000 × 100 /8 = 200,000
Hence, Mrs.santhi’s annual salary in 2016 = 200,000+16,000= Rs.216,000.
Hence, statement-II is only sufficient to answer.
 
8). Answer: b)
From statement I& II,
Let the length of train be x,
Length of the train = (x+120)/10 = x/6
                              = (x+120)/5 = x/3
                             –>5x = 3x +360
                                ∴ x = 360/2 = 180m
∴ Speed of the train = x/6 = 180/6 = 30m/sec
Hence, both statements I& II are sufficient to answer.
 
9). Answer: a)
From both the statements no result is obtained. Hence, both statements I and II together are not sufficient to answer the question.
 
10). Answer: d)
From statement-I,
Required number = 280 × 40/100 × 100/25 = 448
From statement-II,
Let the number be x,
Then, 3x/4 = x – 112
3x = 4x – 448
∴ x = 448

 

Hence, either statement- I or statement-II is sufficient to answer the question. 

 

0 0 votes
Rating
Subscribe
Notify of
guest
0 Comments
Inline Feedbacks
View all comments